Found problems: 14842
2025 Israel TST, P2
A graph with $10^{100}$ vertices satisfies the following condition: Any simple odd cycle has length > 100. Prove there is an independent set in the graph of size at least $\frac{10^{100}}{102}$
2024 TASIMO, 3
$Abdulqodir$ cut out $2024$ congruent regular $n-$gons from a sheet of paper and placed these $n-$gons on the table such that some parts of each of these $n-$gons may be covered by others. We say that a vertex of one of the afore-mentioned $n-$gons is $visible$ if it is not in the interior of another $n-$gon that is placed on top of it. For any $n>2$ determine the minimum possible number of visible vertices. \\
Proposed by David Hrushka, Slovakia
2017 Indonesia MO, 8
A field is made of $2017 \times 2017$ unit squares. Luffy has $k$ gold detectors, which he places on some of the unit squares, then he leaves the area. Sanji then chooses a $1500 \times 1500$ area, then buries a gold coin on each unit square in this area and none other. When Luffy returns, a gold detector beeps if and only if there is a gold coin buried underneath the unit square it's on. It turns out that by an appropriate placement, Luffy will always be able to determine the $1500 \times 1500$ area containing the gold coins by observing the detectors, no matter how Sanji places the gold coins. Determine the minimum value of $k$ in which this is possible.
1998 All-Russian Olympiad Regional Round, 9.4
There is a square of checkered paper measuring $102 \times 102$ squares and a connected figure of unknown shape, consisting of 101 cells. What is the largest number of such figures that can be cut from this square with a guarantee? A figure made up of cells is called [i]connected [/i] if any two its cells can be connected by a chain of its cells in which any two adjacent cells have a common side.
2015 QEDMO 14th, 2
For a natural number $n$ let $W (n)$ be the number of possibilities, to distribute weights with the masses $1, 2,..., n$ all of them between the two bowls of a beam balance so that they are in balance/ Show that $W (100)$ is really larger than $W (99)$.
1996 Estonia National Olympiad, 5
John and Mary play the following game. First they choose integers $n > m > 0$ and put $n$ sweets on an empty table. Then they start to make moves alternately. A move consists of choosing a nonnegative integer $k\le m$ and taking $k$ sweets away from the table (if $k = 0$ , nothing happens in fact). In doing so no value for $k$ can be chosen more than once (by none of the players) or can be greater than the number of sweets at the table at the moment of choice. The game is over when one of the players can make no more moves.
John and Mary decided that at the beginning Mary chooses the numbers $m$ and $n$ and then John determines whether the performer of the last move wins or looses. Can Mary choose $m$ and $n$ in such way that independently of John’s decision she will be able to win?
2022 HMIC, 4
Call a simple graph $G$ [i]quasicolorable[/i] if we can color each edge blue, red, green, or white such that
[list]
[*] for each vertex v of degree 3 in G, the three edges incident to v are either (1) red,
green, and blue, or (2) all white,
[*] not all edges are white.
[/list]
A simple connected graph $G$ has $a$ vertices of degree $4$, $b$ vertices of degree $3$, and no other vertices, where $a$ and $b$ are positive integers. Find the smallest real number $c$ so that the following statement is true: “If $a/b > c$, then $G$ must be quasicolorable.”
2022 Tuymaada Olympiad, 5
Each row of a $24 \times 8$ table contains some permutation of the numbers $1, 2, \cdots , 8.$ In each column the numbers are multiplied. What is the minimum possible sum of all the products?
[i](C. Wu)[/i]
1998 Hungary-Israel Binational, 1
A player is playing the following game. In each turn he flips a coin and guesses the outcome. If his guess is correct, he gains $ 1$ point; otherwise he loses all his points. Initially the player has no points, and plays the game
until he has $ 2$ points.
(a) Find the probability $ p_{n}$ that the game ends after exactly $ n$ flips.
(b) What is the expected number of flips needed to finish the game?
JOM 2025, 3
Minivan and Megavan play a game. For a positive integer $n$, Minivan selects a sequence of integers $a_1,a_2,\ldots,a_n$. An operation on $a_1,a_2,\ldots,a_n$ means selecting an $a_i$ and increasing it by $1$. Minivan and Megavan take turns, with Minivan going first. On Minivan's turn, he performs at most $2025$ operations, and he may choose the same integer repeatedly. On Megavan's turn, he performs exactly $1$ operation instead. Megavan wins if at any point in the game, including in the middle of Minivan's operations, two numbers in the sequence are equal.
[i](Proposed by Ho Janson)[/i]
1964 Leningrad Math Olympiad, grade 8
[b]8.1[/b] Find all primes $p,q$ and $r$ such that $$pqr= 5(p + q + r).$$
[b]8.2 [/b] Prove that if $\overline{ab}/\overline{bc} = a/c$, then $$\overline{abb...bb}/\overline{bb...bbc} = a/c$$ (each number has $n$ digits).
[b]8.3 / 9.1[/b] Construct a triangle with perimeter, altitude and angle at the base.
[b]8.4. / 9.4[/b] Prove that the square of the sum of N distinct non-zero squares of integers is also the sum of $N $squares of non-zero integers.
[b]8.5.[/b] In the quadrilateral $ABCD$ the diagonals $AC$ and $BD$ are drawn. Prove that if the circles inscribed in $ABC$ and $ ADC$ touch each other each other, then the circles inscribed in $BAD$ and in $BCD$ also touch each other.
[b]8.6 / 9.6[/b] If the numbers $A$ and $n$ are coprime, then there are integers $X$ and $Y$ such that $ |X| <\sqrt{n}$, $|Y| <\sqrt{n} $ and $AX-Y$ is divided by $n$. Prove it.
PS. You should use hide for answers.Collected [url=https://artofproblemsolving.com/community/c3983461_1964_leningrad_math_olympiad]here[/url].
2008 South africa National Olympiad, 4
A pack of $2008$ cards, numbered from $1$ to $2008$, is shuffled in order to play a game in which each move has two steps:
(i) the top card is placed at the bottom;
(ii) the new top card is removed.
It turns out that the cards are removed in the order $1,2,\dots,2008$. Which card was at the top before the game started?
2012 Moldova Team Selection Test, 12
Let $k \in \mathbb{N}$. Prove that \[ \binom{k}{0} \cdot (x+k)^k - \binom{k}{1} \cdot (x+k-1)^k+...+(-1)^k \cdot \binom{k}{k} \cdot x^k=k! ,\forall k \in \mathbb{R}\]
2016 Balkan MO Shortlist, C3
The plane is divided into squares by two sets of parallel lines, forming an infinite grid. Each unit square is coloured with one of $1201$ colours so that no rectangle with perimeter $100$ contains two squares of the same colour. Show that no rectangle of size $1\times1201$ or $1201\times1$ contains two squares of the same colour.
[i]Note: Any rectangle is assumed here to have sides contained in the lines of the grid.[/i]
[i](Bulgaria - Nikolay Beluhov)[/i]
2020 Olympic Revenge, 5
Let $n$ be a positive integer. Given $n$ points in the plane, prove that it is possible to draw an angle with measure $\frac{2\pi}{n}$ with vertex as each one of the given points, such that any point in the plane is covered by at least one of the angles.
2019 May Olympiad, 5
There is a board with three rows and $2019$ columns. In the first row are written the numbers integers from $1$ to $2019$ inclusive, ordered from smallest to largest. In the second row, $Ana$ writes those same numbers but ordered at your choice. In each box in the third row write the difference between the two numbers already written in the same column (the largest minus the smallest). $Beto$ have to paint some numbers in the third row so that the sum of the numbers painted is equal to the sum of the numbers in that row that were left unpainted. Can $Ana$ complete the second row so that $Beto$ does not achieve his goal?
2021 BmMT, Ind. Tie
[b]p1.[/b] Isosceles trapezoid $ABCD$ has $AB = 2$, $BC = DA =\sqrt{17}$, and $CD = 4$. Point $E$ lies on $\overline{CD}$ such that $\overline{AE}$ splits $ABCD$ into two polygons of equal area. What is $DE$?
[b]p2.[/b] At the Berkeley Sandwich Parlor, the famous BMT sandwich consists of up to five ingredients between the bread slices. These ingredients can be either bacon, mayo, or tomato, and ingredients of the same type are indistiguishable. If there must be at least one of each ingredient in the sandwich, and the order in which the ingredients are placed in the sandwich matters, how many possible ways are there to prepare a BMT sandwich?
[b]p3.[/b] Three mutually externally tangent circles have radii $2$, $3$, and $3$. A fourth circle, distinct from the other three circles, is tangent to all three other circles. The sum of all possible radii of the fourth circle can be expressed as $\frac{m}{n}$ , where $m$ and $n$ are relatively prime positive integers. Compute $m + n$.
PS. You had better use hide for answers. Collected [url=https://artofproblemsolving.com/community/c5h2760506p24143309]here[/url].
2019 IMO Shortlist, C9
For any two different real numbers $x$ and $y$, we define $D(x,y)$ to be the unique integer $d$ satisfying $2^d\le |x-y| < 2^{d+1}$. Given a set of reals $\mathcal F$, and an element $x\in \mathcal F$, we say that the [i]scales[/i] of $x$ in $\mathcal F$ are the values of $D(x,y)$ for $y\in\mathcal F$ with $x\neq y$. Let $k$ be a given positive integer.
Suppose that each member $x$ of $\mathcal F$ has at most $k$ different scales in $\mathcal F$ (note that these scales may depend on $x$). What is the maximum possible size of $\mathcal F$?
2012 Indonesia MO, 1
Given positive integers $m$ and $n$. Let $P$ and $Q$ be two collections of $m \times n$ numbers of $0$ and $1$, arranged in $m$ rows and $n$ columns. An example of such collections for $m=3$ and $n=4$ is
\[\left[ \begin{array}{cccc}
1 & 1 & 1 & 0 \\
1 & 1 & 0 & 0 \\
0 & 0 & 0 & 0 \end{array} \right].\]
Let those two collections satisfy the following properties:
(i) On each row of $P$, from left to right, the numbers are non-increasing,
(ii) On each column of $Q$, from top to bottom, the numbers are non-increasing,
(iii) The sum of numbers on the row in $P$ equals to the same row in $Q$,
(iv) The sum of numbers on the column in $P$ equals to the same column in $Q$.
Show that the number on row $i$ and column $j$ of $P$ equals to the number on row $i$ and column $j$ of $Q$ for $i=1,2,\dots,m$ and $j=1,2,\dots,n$.
[i]Proposer: Stefanus Lie[/i]
KoMaL A Problems 2021/2022, A. 804
There is a city with $n$ citizens. The city wants to buy [i]sceptervirus[/i] tests with which it is possible to analyze the samples of several people at the same time. The result of a test can be the following:
[list]
[*][i]Virus positive[/i]: there is at least one currently infected person among the people whose samples were analyzed, and none of them were healed from an earlier infection.
[*][i]Antibody positive[/i]: there is at least one person who was healed from an earlier infection among the people whose samples were analyzed, and none of them are infected now.
[*][i]Neutral[/i]: either all of the people whose samples were analyzed are not infected, or there is at least one currently infected person and one person who was healed from an earlier infection. (Viruses and antibodies in samples completely neutralize each other.)
[/list]
What is the smallest number of tests to buy if we would like to know if the sceptervirus is present now or it has been present earlier? (The samples are taken from the people at the same time. The people are either infected now, have been infected earlier, or haven't contacted the virus yet.)
[i]Proposed by Csongor Beke, Cambridge[/i]
2021 VIASM Math Olympiad Test, Problem 2
Given a square $5$ x $7$ board and $35$ pieces, each piece is formed by $3$ squares like below:
[size=75][center][img]https://i.ibb.co/hFDhp9p/Screenshot-2023-03-26-061057.png[/img][/center][/size]
Can we fill the board with $35$ pieces such that there are exactly $3$ pieces superimpose on every square of the given board?
[i][color=#E06666]Note: we can rotate, turn upside down the pieces[/color][/i]
2021 Korea Junior Math Olympiad, 6
In a meeting of $4042$ people, there are $2021$ couples, each consisting of two people. Suppose that $A$ and $B$, in the meeting, are friends when they know each other. For a positive integer $n$, each people chooses an integer from $-n$ to $n$ so that the following conditions hold. (Two or more people may choose the same number).
[list]
[*] Two or less people chose $0$, and if exactly two people chose $0$, they are coupled.
[*] Two people are either coupled or don't know each other if they chose the same number.
[*] Two people are either coupled or know each other if they chose two numbers that sum to $0$.
[/list]
Determine the least possible value of $n$ for which such number selecting is always possible.
2023 May Olympiad, 5
On the table there are $50$ stacks of coins that have $1,2,3,…,50$ coins respectively. Ana and Beto play the following game in turns:
First, Ana chooses one of the $50$ piles on the table, and Beto decides if that pile is for Ana or for him.
Then, Beto chooses one of the $49$ remaining piles on the table, and Ana decides if that pile is for her or for Beto.
They continue playing alternately in this way until one of the players has $25$ batteries.
When that happens, the other player takes all the remaining stacks on the table and whoever has the most coins wins.
Determine which of the two players has a winning strategy.
2019 Caucasus Mathematical Olympiad, 1
Pasha placed numbers from 1 to 100 in the cells of the square $10\times 10$, each number exactly once. After that, Dima considered all sorts of squares, with the sides going along the grid lines, consisting of more than one cell, and painted in green the largest number in each such square (one number could be colored many times). Is it possible that all two-digit numbers are painted green?
2023 Pan-American Girls’ Mathematical Olympiad, 2
In each cell of an \(n \times n\) grid, one of the numbers \(0\), \(1,\) or \(2\) must be written. Determine all positive integers \(n\) for which there exists a way to fill the \(n \times n\) grid such that, when calculating the sum of the numbers in each row and each column, the numbers \(1, 2, \ldots, 2n\) are obtained in some order.